Sei sulla pagina 1di 5

MAT2125 Winter 2018

Assignment 2
Due Thursday February 1, 11:30 AM, in the assignment drop box in
King Edward 585.

Each question is worth 10 marks.

1. Do Exercise 2.2.5.

2. Let (an )∞ ∞
n=1 be a bounded sequence and let (bn )n=1 be a sequence that
diverges to ∞. Prove that (an + bn )∞
n=1 diverges to ∞.

3. Let (an )∞ ∞ ∞
n=1 and (bn )n=1 be bounded sequences, and suppose that (an )n=1
converges. Prove that
   
lim sup(an + bn ) = lim an + lim sup bn .
n→∞ n→∞ n→∞

4. Let (an )∞
n=1 be a bounded sequence and let L ∈ R. Suppose that for every
subsequence (ank )∞k=1 , either

lim ank = L
k→∞

or (ank )∞ ∞
k=1 does not converge. Prove that (an )n=1 converges to L.
[Hint. Proof by contradiction.]

5. [TBB08, Exercise 2.14.17] Let (an )∞n=1 be a bounded sequence. Suppose



that the sequence (an + 2an+1 )n=1 converges to 3L ∈ R. Prove that (an )∞ n=1
converges to L.
[Hint. Show that if |an + 2an+1 − 3L| < η then |an+1 − L| < |an2−L| + η2 . Then
iterate this inequality.]

Bonus (Up to 5 bonus marks) Do Question 5 without assuming that (an )∞


n=1
is bounded.
Solutions.
1. (The question is: let (an )∞
√ be a sequence with an ≥ 0 for all n, converg-
n=1

ing to L ≥ 0. Show an → L as n → ∞.)
Let (an )∞ √be a sequence with an ≥ 0 for all n, converging to L ≥ 0. To
n=1

show an → L, consider two cases.
√ Case 1. L > 0. Given  > 0, choose n0 such that for n ≥ n0 , |an − L| <
 L. (2 marks)
Then for n ≥ n0 ,

√ √ a −L
n
| an − L| = √ √ (1 mark)

an + L
|an − L|
≤ √ (1 mark)
L
< , (1 mark)
as required.
Case 2 L = 0. Given  > 0, choose n0 such that for n ≥ n0 , |an | =
|an − L| < 2 . (2 marks)
Then for n ≥ n0 ,
√ √ √
| an − L| = an (1 mark)

< 2 =  (2 marks)
as required.

2. Let M be a lower bound on (an )∞ n=1 , so that we have an ≥ M for all


n ∈ N≥1 . (3 marks)
Given R > 0, choose n0 such that for all n ≥ n0 ,
bn > R − M. (3 marks)
Then for n ≥ n0 ,
an + bn > M + R − M = R,
as required. (4 marks)

3. We will show an inequality in each direction. Set L1 := limn→∞ an and


L2 := lim supn→∞ bn .
Let  > 0. Then there exists n0 such that for all n ≥ n0 ,
 
an < L1 + and bn < L2 + . (1 mark)
2 2
Hence for n ≥ n0 ,
 
an + bn < L1 + + L2 + = L1 + L2 + . (1 mark)
2 2
Thus, L1 + L2 +  is an eventual upper bound for (an + bn )∞
n=1 , so

lim sup(an + bn ) ≤ L1 + L2 + . (1 mark)


n→∞

Since  > 0 is arbitrary, we conclude that


   
lim sup(an + bn ) ≤ lim an + lim sup bn . (2 marks)
n→∞ n→∞ n→∞

For the other inequality, we set L := lim supn→∞ an + bn . Let  > 0. Then
there exists n0 such that for all n ≥ n0 ,
 
an > L1 − and an + bn < L + . (1 mark*)
2 2
Hence for n ≥ n0 ,
 
b n = an + b n − an < L + − (L1 − ) = L − L1 + . (1 mark*)
2 2
Thus, L − L1 +  is an eventual upper bound for (bn )∞
n=1 , so

lim sup(bn ) ≤ L − L1 + . (1 mark*)


n→∞

Since  > 0 is arbitrary, we conclude that


 
lim sup bn ≤ lim sup(an + bn ) − lim an ,
n→∞ n→∞ n→∞

which rearranges as
   
lim sup(an + bn ) ≥ lim an + lim sup bn . (2 marks)
n→∞ n→∞ n→∞
Alternate solutions.
One may quote Exercise 2.7.4 (a) for ≤. For this, you must note that
lim supn→∞ an = limn→∞ an (by Theorem 2.7.6). In this case, 2 marks are
given for ≤, and each (1 mark*) is changed to to (2 marks).
For ≥, here is an alternate solution. Observe that applying the inequality
≤ with −an in place of an and an + bn in place of an , we get (1 mark*)
lim sup bn = lim sup((−an ) + an + bn ) (1 mark*)
n→∞ n→∞
   
≤ lim −an + lim sup an + bn . (1 mark*)
n→∞ n→∞

Since limn→∞ −an = − limn→∞ an , we may rearrange to get


   
lim an + lim sup bn ≤ lim sup(an + bn ),
n→∞ n→∞ n→∞

as required. (2 marks)

4. Suppose for a contradiction that (an )∞ n=1 does not converge to L. This
means that there is some  > 0 such that, for all n0 ∈ N≥1 there exists n ≥ n0
such that
(2 marks)
|an − L| ≥ .
Using this, we may define a subsequence (ank )∞ k=1 such that |ank − L| ≥ 
for all k. (First with n0 = 1, we can find n1 ≥ 1 such that |an1 − L| ≥ ; then
using n1 + 1 in place of n0 , we get some n2 > n1 such that |an2 − L| ≥ , and
so on.) (2 marks)
∞ ∞
Since (an )n=1 is bounded, so is (ank )k=1 . (1 mark)
Hence by the Bolzano–Weierstrass Theorem, it has a convergent subse-
quence (ankl )∞l=1 . (2 marks)

This is itself a subsequence of (an )n=1 , however since |ankl − L| ≥  for all
l, we also have
| lim ankl − L| ≥ . (1 mark)
l→∞
This contradicts the hypothesis that the limit of this subsequence is L.
(2 marks)

5. Let  > 0 be given. Since (an )∞


n=1 is bounded, let M > 0 be such that
|an − L| ≤ M for all n ∈ N≥1 . Choose m0 such that for all n ≥ m0 ,

|an + 2an+1 − 3L| < . (2 marks)
2
Then for n ≥ m0 , since an+1 − L = 12 (an + 2an+1 − 3L − (an − L)), we
have
1 1
|an+1 − L| ≤ |an + 2an+1 − 3L| + |an − L|
2 2
 1
< + |an − L|. (2 marks)
4 2
Iterating this, we obtain (using induction)

 1  M
|am0 +1 − L| < + |am0 − L| ≤ + ,
4 2 2 2
  M
|am0 +2 − L| < + + ,
4 8 4
..
.
   M
|am0 +k − L| < + + · · · + k+1 + k
4 8 2 2
 M
< + k. (2 marks)
2 2
Pick n0 ≥ m0 such that 2n0 −m0 < 2 .
M
(2 marks)
M 
Then for n ≥ n0 , since we may write n = m0 + k where 2k
< we get
2
,
 
|an − L| < + = ,
2 2
as required. (2 marks)

Potrebbero piacerti anche